Symmetric Expression with a twist

Algebra Level 3

Consider all pairs of real numbers such that a + b = 3 a + b = 3 .

What is the minimum value of

a 2 b 2 2 a 2 b 2 a b 2 + a 2 + 4 a b + b 2 2 a 2 b + 1 ? a^2 b^2 - 2a^2 b - 2 a b^2 + a^2 + 4ab + b^2 - 2a - 2b + 1?

1 0 0.25 6.25E-2

This section requires Javascript.
You are seeing this because something didn't load right. We suggest you, (a) try refreshing the page, (b) enabling javascript if it is disabled on your browser and, finally, (c) loading the non-javascript version of this page . We're sorry about the hassle.

5 solutions

Krishna Sharma
Nov 20, 2014

The expression can be written as

a 2 b 2 2 a 2 b 2 a b 2 + 2 a b + ( a + b 1 ) 2 a^{2}b^{2} -2a^{2}b - 2ab^{2} + 2ab + (a + b -1)^{2}

Since a + b = 3

a 2 b 2 2 a b ( a + b 1 ) + 4 a^{2}b^{2} -2ab( a + b -1) + 4

a 2 b 2 4 a b + 4 a^{2}b^{2} - 4ab + 4

Now let a b = t ab = t

Differentiating the function

t 2 4 t + 4 t^{2} - 4t + 4

we get

2t - 4 = 0

t = 2 t = 2

we have to check if ab = 2 valid ( given a + b = 3)

For (a,b) = {1,2} it satisfies the given condition hence

a b m i n = 2 ab_{min} = 2

a 2 b 2 4 a b + 4 = 0 a^{2}b^{2} - 4ab + 4 = \boxed{0}

Calvin Lin Staff
Nov 20, 2014

The expression is equal to ( a 1 ) 2 ( b 1 ) 2 (a-1)^2 ( b-1)^ 2 .

The minimum is clearly 0, and occurs when { a , b } = { 1 , 2 } \{ a, b \} = \{ 1, 2 \} .


Note: Some may think that the minimum occurs when a = b = 3 2 a = b = \frac{3}{2} , which will yield the value 0.0625. Why doesn't this work?

a 3 + b 3 = 1 \frac{a}{3} + \frac{b}{3} = 1

A triangle is formed ,

A r e a = f ( a ) = 1 2 a ( 3 a ) Area =f(a) = \frac{1}{2}a(3-a)

f ( a ) = 3 2 a f'(a) = 3 - 2a

f ( a ) = 0 , a = 3 2 f'(a)= 0 , a = \frac{3}{2}

checking whether it is the point of maxima of minima ,

f ( a ) = 2 f''(a) = -2 thus for a = 3 2 a = \frac{3}{2} , the area obtained is maximum. That's why it does'nt work here.

Sir how would we find the point of minima using derivatives?

sandeep Rathod - 6 years, 6 months ago

I took the expression to (ab-2)^{2} and the a=1 or b=2 which gave the minimum at 0.

Rajarshi Chatterjee - 6 years, 6 months ago

Log in to reply

Right, because ( a 1 ) ( b 1 ) = a b a b + 1 = a b 2 (a-1)(b-1) = ab - a - b + 1 = ab - 2 .

Calvin Lin Staff - 6 years, 6 months ago

Log in to reply

Thank you!!

Rajarshi Chatterjee - 6 years, 6 months ago
Jubayer Nirjhor
Nov 21, 2014

Suppose n = a + b = 3 n=a+b=3 and m = a b m=ab . Then the given is m 2 2 m n + n 2 + 2 m 2 n + 1. m^2-2mn+n^2+2m-2n+1. Subbing n = 3 n=3 we end up with m 2 4 m + 4 = ( m 2 ) 2 . m^2-4m+4=(m-2)^2. Since m n 2 / 4 = 2.25 m\le n^2/4=2.25 by AM-GM, we can take m = 2 m=2 which yields the minimum, 0 0 .

Equality occurs at the roots of x 2 3 x + 2 x^2-3x+2 by Vieta's, which are 1 , 2 1,2 .

Hareesh Batchu
Nov 23, 2014

a=1;b=2 then slove

Qwswhejjnjhhbwjjkkskk

Priti Gupta - 4 years, 4 months ago
Aareyan Manzoor
Nov 23, 2014

a 2 b 2 2 a b ( a + b ) + ( a + b ) 2 2 a b + 4 a b 2 ( a + b ) + 1 a^2b^2 -2ab(a+b)+(a+b)^2 -2ab+4ab-2(a+b)+1 ( a b ) 2 2 a b ( 3 ) + 9 + 2 a b 6 + 1 (ab)^2 -2ab(3)+9+2ab-6+1 the minimum value of ab >0 is a+b-1=2{note that zero wont work} 2 2 4 × 3 + 9 + 4 + 1 6 2^2 -4\times 3 +9+4+1-6 4 12 + 14 6 4-12+14-6 0 \boxed{0}

0 pending reports

×

Problem Loading...

Note Loading...

Set Loading...